Difference between revisions of "2024 AIME II Problems/Problem 12"
(→Solution 1 (completely no calculus required)) |
|||
Line 50: | Line 50: | ||
This is easily factored, allowing us to determine that <math>x=\frac{1}{8},\frac{1}{2}</math>. The latter root is not our answer, since on line <math>\overline{AB}</math>, <math>y(\frac{1}{2})=0</math>, the horizontal line segment running from <math>(0,0)</math> to <math>(1,0)</math> covers that point. From this, we see that <math>x=\frac{1}{8}</math> is the only possible candidate. | This is easily factored, allowing us to determine that <math>x=\frac{1}{8},\frac{1}{2}</math>. The latter root is not our answer, since on line <math>\overline{AB}</math>, <math>y(\frac{1}{2})=0</math>, the horizontal line segment running from <math>(0,0)</math> to <math>(1,0)</math> covers that point. From this, we see that <math>x=\frac{1}{8}</math> is the only possible candidate. | ||
− | Going back to line <math>\overline{AB}, y= -\sqrt{3}x+\frac{\sqrt{3}}{2}</math>, plugging in <math>x=\frac{1}{8} </math> yields <math>y=\frac{ | + | Going back to line <math>\overline{AB}, y= -\sqrt{3}x+\frac{\sqrt{3}}{2}</math>, plugging in <math>x=\frac{1}{8} </math> yields <math>y=\frac{3\sqrt{3}}{8}</math>. The distance from the origin is then given by <math>\sqrt{\frac{1}{8^2}+(\frac{2\sqrt{3}}{8})^2} =\sqrt{\frac{7}{16}}</math>. That number squared is <math>\frac{7}{16}</math>, so the answer is <math>\boxed{023}</math>. |
Revision as of 10:31, 14 May 2024
Contents
Problem
Let \(O=(0,0)\), \(A=\left(\tfrac{1}{2},0\right)\), and \(B=\left(0,\tfrac{\sqrt{3}}{2}\right)\) be points in the coordinate plane. Let \(\mathcal{F}\) be the family of segments \(\overline{PQ}\) of unit length lying in the first quadrant with \(P\) on the \(x\)-axis and \(Q\) on the \(y\)-axis. There is a unique point \(C\) on \(\overline{AB}\), distinct from \(A\) and \(B\), that does not belong to any segment from \(\mathcal{F}\) other than \(\overline{AB}\). Then \(OC^2=\tfrac{p}{q}\), where \(p\) and \(q\) are relatively prime positive integers. Find \(p+q\).
Solution 1 (completely no calculus required)
Begin by finding the equation of the line : Now, consider the general equation of all lines that belong to . Let be located at and be located at . With these assumptions, we may arrive at the equation . However, a critical condition that must be satisfied by our parameters is that , since the length of .
Here's the golden trick that resolves the problem: we wish to find some point along such that passes through iff . It's not hard to convince oneself of this, since the property implies that if , then .
We should now try to relate the point to some value of . This is accomplished by finding the intersection of two lines:
Where we have also used the fact that , which follows nicely from .
Square both sides and go through some algebraic manipulations to arrive at
Note how is a solution to this polynomial, and it is logically so. If we found the set of intersections consisting of line segment with an identical copy of itself, every single point on the line (all values) should satisfy the equation. Thus, we can perform polynomial division to eliminate the extraneous solution .
Remember our original goal. It was to find an value such that is the only valid solution. Therefore, we can actually plug in back into the equation to look for values of such that the relation is satisfied, then eliminate undesirable answers. This is easily factored, allowing us to determine that . The latter root is not our answer, since on line , , the horizontal line segment running from to covers that point. From this, we see that is the only possible candidate.
Going back to line , plugging in yields . The distance from the origin is then given by . That number squared is , so the answer is .
~Installhelp_hex
Solution 2
Now, we want to find . By L'Hôpital's rule, we get . This means that , so we get .
~Bluesoul
Solution 3
The equation of line is
The position of line can be characterized by , denoted as . Thus, the equation of line is
Solving (1) and (2), the -coordinate of the intersecting point of lines and satisfies the following equation:
We denote the L.H.S. as .
We observe that for all . Therefore, the point that this problem asks us to find can be equivalently stated in the following way:
We interpret Equation (1) as a parameterized equation that is a tuning parameter and is a variable that shall be solved and expressed in terms of . In Equation (1), there exists a unique , denoted as (-coordinate of point ), such that the only solution is . For all other , there are more than one solutions with one solution and at least another solution.
Given that function is differentiable, the above condition is equivalent to the first-order-condition
Calculating derivatives in this equation, we get
By solving this equation, we get
Plugging this into Equation (1), we get the -coordinate of point :
Therefore, \begin{align*} OC^2 & = x_C^2 + y_C^2 \\ & = \frac{7}{16} . \end{align*}
Therefore, the answer is .
~Steven Chen (Professor Chen Education Palace, www.professorchenedu.com)
Solution 4 (coordinate bash)
Let be a segment in with x-intercept and y-intercept . We can write as \begin{align*} \frac{x}{a} + \frac{y}{b} &= 1 \\ y &= b(1 - \frac{x}{a}). \end{align*} Let the unique point in the first quadrant lie on and no other segment in . We can find by solving and taking the limit as . Since has length , by the Pythagorean theorem. Solving this for , we get \begin{align*} a^2 + b^2 &= 1 \\ b^2 &= 1 - a^2 \\ \frac{db^2}{da} &= \frac{d(1 - a^2)}{da} \\ 2a\frac{db}{da} &= -2a \\ db &= -\frac{a}{b}da. \end{align*} After we substitute , the equation for becomes
In , and . To find the x-coordinate of , we substitute these into the equation for and get \begin{align*} \frac{\sqrt{3}}{2}(1 - \frac{x}{\frac{1}{2}}) &= (\frac{\sqrt{3}}{2} - \frac{\frac{1}{2}}{\frac{\sqrt{3}}{2}} da)(1 - \frac{x}{\frac{1}{2} + da}) \\ \frac{\sqrt{3}}{2}(1 - 2x) &= (\frac{\sqrt{3}}{2} - \frac{da}{\sqrt{3}})(1 - \frac{x}{\frac{1 + 2da}{2}}) \\ \frac{\sqrt{3}}{2} - \sqrt{3}x &= \frac{3 - 2da}{2\sqrt{3}}(1 - \frac{2x}{1 + 2da}) \\ \frac{\sqrt{3}}{2} - \sqrt{3}x &= \frac{3 - 2da}{2\sqrt{3}} \cdot \frac{1 + 2da - 2x}{1 + 2da} \\ \frac{\sqrt{3}}{2} - \sqrt{3}x &= \frac{3 + 6da - 6x - 2da - 4da^2 + 4xda}{2\sqrt{3} + 4\sqrt{3}da} \\ (\frac{\sqrt{3}}{2} - \sqrt{3}x)(2\sqrt{3} + 4\sqrt{3}da) &= 3 + 6da - 6x - 2da - 4da^2 + 4xda \\ 3 + 6da - 6x - 12xda &= 3 + 4da - 6x - 4da^2 + 4xda \\ 2da &= -4da^2 + 16xda \\ 16xda &= 2da + 4da^2 \\ x &= \frac{da + 2da^2}{8da}. \end{align*} We take the limit as to get We substitute into the equation for to find the y-coordinate of : The problem asks for so .
Solution 5 (small perturb)
Let's move a little bit from to , then must move to to keep . intersects with at . Pick points and on and such that , , we have . Since is very small, , , so , , by similarity, . So the coordinates of is .
so , the answer is .
Video Solution
https://youtu.be/914687Yv6SY?si=tc6XfoOIHu0gu6AL
(no calculus)
~MathProblemSolvingSkills.com
Video Solution
~Steven Chen (Professor Chen Education Palace, www.professorchenedu.com)
Query
Let be a fixed point in the first quadrant. Let be a point on the positive -axis and be a point on the positive -axis such that passes through and the length of is minimal. Let be the point such that is a rectangle. Prove that . (One can solve this through algebra/calculus bash, but I'm trying to find a solution that mainly uses geometry. If you know such a solution, write it here on this wiki page.) ~Furaken
I think there is such a geometry way: Let pass through while point is on the outside of line segment and point is in between and . We aim to show is longer than . Now since is the altitude of triangle yet just a cevian on the base of triangle (thus making the height shorter than ), it suffices to show the area of triangle is bigger than that of triangle . To do this, we compare these two triangles (let intersect at point ), and we just want to show . This is trivial by similarity ratios. ~gougutheorem
Thanks! Now we know that it's possible to solve the AIME problem with only geometry. ~Furaken
See also
2024 AIME II (Problems • Answer Key • Resources) | ||
Preceded by Problem 11 |
Followed by Problem 13 | |
1 • 2 • 3 • 4 • 5 • 6 • 7 • 8 • 9 • 10 • 11 • 12 • 13 • 14 • 15 | ||
All AIME Problems and Solutions |
The problems on this page are copyrighted by the Mathematical Association of America's American Mathematics Competitions.